Lista de exercícios do ensino médio para impressão
(MACKENZIE - 1977) O gráfico abaixo pode ser da função:
gráfico de um menos seno de x

a)
$|senx|$
b)
$sen^2x$
c)
$1-|senx|$
d)
$1-|cosx|$
e)
não sei.


 



resposta: Alternativa C
×
(CESCEM - 1975) A função que melhor se adapta ao gráfico abaixo é:
a)
$y = sen(\dfrac{x}{2})$
b)
$y = cos(\dfrac{x}{2})$
c)
$y = sen{2x}$
d)
$y = cos{2x}$
e)
$y = sen{x}$
gráfico de função

 



resposta: Alternativa A
×
(UFGO) Simplificando a expressão $\,\dfrac{\operatorname{tg}a\,+\,\operatorname{tg}b}{\operatorname{cotg}a\,+\,\operatorname{cotg}b}\,$, obtém-se:
a)
$\,\operatorname{tg}a \centerdot \operatorname{tg}b\,$
b)
$\,\operatorname{cotg}a \centerdot \operatorname{cotg}b\,$
c)
$\,\operatorname{tg}(a\,+\,b)\,$
d)
$\,\operatorname{cotag}(a\,+\,b)\,$
e)
$\,\operatorname{tg}a \centerdot \operatorname{cotg}b\,$

 



resposta: Alternativa A
×
(VUNESP - 1990) Uma gangorra é formada por uma haste rígida AB , apoiada sobre uma mureta de concreto no ponto C , como na figura. As dimensões são:$\;\overline{AC}\,=\,1,2\;$m, $\;\overline{CB}\,=\,1,8\;$m, $\;\overline{DC}\,=\,\overline{CE}\,=\,\overline{DE}\,=\,1\;$m. Quando a extremidade B da haste toca o chão, a altura da extremidade A em relação ao chão é:
a)
$\sqrt{3}\;$m
b)
$ \dfrac{3}{ \sqrt{3}}\;$m
c)
$\dfrac{6 \sqrt{3}}{5}\;$m
d)
$\dfrac{5 \sqrt{3}}{6}\;$m
e)
$2\sqrt{2}\;$m
gangorra

 



resposta:
gangorra da vunesp

Considerações:

A figura representa a situação descrita no enunciado, com o ponto B tocando o chão.

A distância $\;\overline{PC}\;$ é a altura da mureta, cuja secção é um triângulo equilátero de lado medindo 1 metro, portanto $\;\overline{PC}\;$ vale $\;1\centerdot\dfrac{\sqrt{3}}{2}\phantom{X}$ (veja altura do triângulo equilátero em função do lado neste exercício
Resolução:
O triângulo $\;AQB\;$ é semelhante ao triângulo $\;CPB\;$ pois possuem o ângulo $\;\hat{B}\;$ comum e os ângulos $\;\hat{P}\;$ e $\;\hat{Q}\;$ são ângulos retos. Como são triângulos semelhantes, seus lados são proporcionais.
$\;\dfrac{\overline{AB}}{\overline{CB}}\,=\,\dfrac{\overline{AQ}}{\overline{CP}}\;\Rightarrow\;$
$\;\dfrac{1,2\, +\, 1,8}{1,8}\,=\,\dfrac{H}{\frac{\sqrt{3}}{2}}\;\Rightarrow\;$ $\;H\,=\,\dfrac{\sqrt{3}}{2}\centerdot\dfrac{30}{18}\;\Rightarrow\;$
$\;H\,=\,\dfrac{\sqrt{3}}{1}\centerdot\dfrac{15}{18}\;\Rightarrow\;$
$\;H\,=\,\dfrac{5\sqrt{3}}{6}\;\Rightarrow\;$ corresponde à
Alternativa D

×
"Quando percebi que o doente expirava, recuei aterrado, e dei um grito, mas ninguém me ouviu."(M. de Assis)
A função sintática das palavras doente — grito — ninguém — me é, respectivamente:
a)
sujeito, objeto direto, objeto direto, objeto indireto
b)
objeto direto, sujeito, objeto direto, sujeito
c)
sujeito, objeto indireto, sujeito, objeto direto
d)
objeto indireto, objeto direto, sujeito, objeto direto
e)
sujeito, objeto direto, sujeito, objeto direto

 



resposta: Alternativa E
×
Assinale a alternativa em que o A craseado introduz termo sintático com função de objeto indireto.
a)
Ele se referia à mesma pessoa.
b)
Quando se dirigirá à casa paterna?
c)
Chegaremos possivelmente às duas horas.
d)
Estamos à sua espera.
e)
Foi um almoço à moda americana.

 



resposta: Alternativa A
×
Assinale a alternativa em que o A craseado introduz termo sintático com função de objeto indireto.

a) Ele se referia à mesma pessoa.
b) Quando se dirigirá à casa paterna?
c) Chegaremos possivelmente às duas horas.
d) Estamos à sua espera.
e) Foi um almoço à moda americana.

 



resposta: A
×
Observe os termos grifados:
Ao bom ladrão, respondeu-lhe o Cristo prometendo o Paraíso.
Sua função é:
a)
objeto direto pleonástico.
b)
objeto direto preposicionado e objeto indireto.
c)
adjunto adverbial e objeto indireto.
d)
objeto indireto pleonástico.
e)
adjunto adnominal.

 



resposta: Alternativa D
×
(ITA - 2004) Considerando as funções

$\phantom{XX} \operatorname{arc\,sen:}[-1, +1] \rightarrow [ - \pi / 2, \pi / 2 ]\phantom{XX}$ e $\phantom{XXX} \operatorname{arc\,cos:} [-1, + 1] \rightarrow [0, \pi[\phantom{XX}$ ,

assinale o valor de $\phantom{X}cos(\operatorname{arc\,sen} \dfrac{3}{5} + \operatorname{arc\,cos} \dfrac{4}{5})\phantom{X}$.

a)
$\;\dfrac{6}{25}\;$
b)
$\;\dfrac{7}{25}\;$
c)
$\;\dfrac{1}{3}\;$
d)
$\;\dfrac{2}{5}\;$
e)
$\;\dfrac{5}{12}\;$

 



resposta: Alternativa B
×
Considerando a oração "os cabelos, em bandós, eram apanhados sobre a nuca por um velho pente de tartaruga" , assinale a opção em que a função sintática dos constituintes grifados aparece correta e na ordem correta.
a)
núcleo do agente da passiva, adjunto adnominal
b)
núcleo do agente da passiva, complemento nominal
c)
núcleo do adjunto adverbial de lugar, adjunto adnominal
d)
núcleo do adjunto adverbial de lugar, complemento nominal
e)
núcleo do agente da passiva, adjunto nominal

 



resposta: Alternativa A
×
Leia o trecho abaixo e indique a função sintática das palavras grifadas.
" Como é solene e grave, no meio das nossas matas, a hora misteriosa do crepúsculo, em que a natureza se ajoelha aos pés do criador, para murmurar a prece da noite." (J. de Alencar. O Guarani.)
As funções sintáticas das palavras sublinhadas são, respectivamente:
a)
predicativo do sujeito, adjunto adverbial de lugar, núcleo do sujeito, adjunto adnominal, sujeito, adjunto adverbial de lugar, objeto direto e adjunto adnominal.
b)
adjunto adnominal, predicativo do sujeito, objeto direto, sujeito, agente da passiva, adjunto adverbial de lugar, sujeito e adjunto adnominal.
c)
sujeito, sujeito, adjunto adverbial de lugar, predicativo do sujeito, objeto direto, objeto indireto, complemento nominal e sujeito.
d)
objeto direto, agente da passiva, sujeito, objeto indireto, complemento nominal, sujeito, adjunto adverbial de lugar e sujeito.
e)
agente da passiva, adjunto adnominal, sujeito, sujeito, predicativo do sujeito, complemento nominal, objeto direto e objeto indireto.

 



resposta: (A)
×
Construir os gráficos cartesianos das seguintes funções exponenciais:

a) $y\;=\;3^x$

b) $y\;=\;(\frac{1}{3})^x$

c) $y\;=\;4^x$

d) $y\;=\;10^x$

e) $y\;=\;10^{-x}$

f) $y\;=\;(\frac{1}{e})^x$

 



resposta:
a) $\;y\,=\,3^{\large x}\;$
gráfico cartesiano da função exponencial 3 elevado a x
b) $\;y\,=\,\left(\dfrac{1}{3}\right)^{\large x}\;$
gráfico da função y igual um terço elevado a x
c) $\;y\,=\,4^{\large x}\;$
gráfico cartesiano da função exponencial 4 elevado a x

×
Contruir o gráfico cartesiano da função em $\;\mathbb{R}\;$ definida por $\;f(x)\,=\,2^{2x - 1}$

 



resposta:
×
Construir os gráficos das funções em $\;\mathbb{R}\;$ definidas por:
a)
$f(x)\;=\;2^{\large 1-x}$
b)
$f(x)\;=\;3^{\large \frac{x + 1}{2}}$
c)
$f(x)\;=\;2^{\large |x|}$
d)
$f(x)\;=\;({\large\frac{1}{2}})^{\large 2x + 1}$
e)
$f(x)\;=\;({\large\frac{1}{2}})^{\large |x|}$

 



resposta:
a)
gráfico da função 2 elevado a 1 menos x
b)
gráfico da função f de x igual a 3 elevado à fração x + 1 sobre 2
c)
d)
e)

×
Construir os gráficos das funções em $\;{\rm I\!R}\;$ definidas por:
a)
$\;{\large f(x)\;=\;2^{x}\;+\;2^{-x}}$
b)
$\;{\large f(x)\;=\;2^{x}\;-\;2^{-x}}$

 



resposta:
×
(ITA - 2004) Considere a função $\;f : {\rm I\!R} \rightarrow \mathbb{C}$, $f(x) = 2\;cosx + 2\;i\;senx$. Então, $\;\forall \; x, y \; \in \; {\rm I\!R}\;$, o valor do produto $\;f(x)f(y)\;$ é igual a:
a)
$f(x + y)$
b)
$2f(x + y)$
c)
$4\;i\;f(x + y)$
d)
$f(xy)$
e)
$2f(x) + 2\;i\;f(y)$

 



resposta: (B)
×
(ITA - 2004) Considere um cilindro circular reto, de volume igual a $\;360 \pi \; cm^3\;$, e uma pirâmide regular cuja base hexagonal está inscrita na base do cilindro. Sabendo que a altura da pirâmide é o dobro da altura do cilindro e que a área da base da pirâmide é de $\;54\sqrt{3}\;cm^2\;$, então, a área lateral da pirâmide mede, em $cm^2$,
a)
$\;18\sqrt{427}$
b)
$\;27\sqrt{427}$
c)
$\;36\sqrt{427}$
d)
$\;108\sqrt{3}$
e)
$\;45\sqrt{427}$

 



resposta:
hexágono regular inscrito na circunferência
Considerações:
Observe a figura que representa um hexágono regular inscrito numa circunferência:
1. o hexágono regular é formado por 6 triângulos equiláteros de lado igual ao raio da circunferência R.
2. a altura $\;h\;$ de cada triângulo equilátero em função do seu lado $\;R\;$ é $\;\dfrac{R\sqrt{3}}{2}\;$(veja esse exercício).
3.Então a área de cada triângulo equilátero é base × altura ÷ 2
$\;\rightarrow\;\dfrac{R\times h}{2}\;=\;\dfrac{R\times \frac{R\sqrt{3}}{2}}{2}\;=\;\dfrac{R^{\large 2}\sqrt{3}}{4}\;$ e a área do hexágono é $\;\rightarrow\;S_H\;=\,6\centerdot\dfrac{R^{\large 2}\sqrt{3}}{4}\;$

pirâmide hexagonal
Resolução:
Conforme o enunciado, a base da pirâmide tem área $\;54\sqrt{3}\,cm^2\;$
1. calcular $\;R\;$:
$\;S_H\;=\,6\centerdot\dfrac{R^{\large 2}\sqrt{3}}{4}\;=\;54\sqrt{3} \Rightarrow \;R^{\large 2}\,=\,36\;\Rightarrow\;R\,=\,6\;$cm
2. calcular a altura da pirâmide $\;H\;$:
A altura da pirâmide é o dobro da altura do cilindro. Se a altura da pirâmide é $\;H\;$, então a altura do cilindro é $\;\dfrac{H}{2}\;$.
O volume do cilindro é Área da base × altura e conforme o enunciado vale $\;360\pi\,cm^3\;$.$\;\pi\centerdot R^{\large2}\centerdot \dfrac{H}{2}\,=\,360\pi\;\Rightarrow \;H\,=\,20\,cm\;$
3. Calcular a altura de uma face da pirâmide ($\;\overline{VM}\;$):
Observe na figura a pirâmide. Traçando-se a altura de uma das faces da pirâmide, temos o segmento $\;\overline{VM}\;$, que define o triângulo retângulo $\;VOM\;$ reto no ângulo $\;\hat{O}\;$.
Pelo Teorema de Pitágoras:
$\,\left\{\begin{array}{rcr} \mbox{cateto}\; \overline{OM}\; \longrightarrow \dfrac{R\sqrt{3}}{2}\;=\;3\sqrt{3} & \\ \mbox{cateto}\;\overline{OV}\; \longrightarrow\;\phantom{XX}\;H\,= 20\phantom{X} & \\ \end{array} \right.\,$
$\;(VM)^{\large 2}\,=\,(OM)^{\large 2}\,+\,(OV)^{\large 2}\;\Rightarrow\;$ $\,(VM)^{\large 2}\,=\,(3\sqrt{3})^{\large 2}\,+\,20^{\large 2}\;=\;27\,+\,400\,=\,427\;\Rightarrow\;$ $\, \overline{VM}\,=\,\sqrt{427}\;$
4. Calcular a área lateral da pirâmide:
A área de uma face da pirâmide é $\;\overline{AB}\centerdot\overline{VM}\div 2\;$ $=\,\dfrac{R\centerdot\overline{VM}}{2}\;=\;\dfrac{6\times\sqrt{427}}{2}\;=\,3\sqrt{427};$A área lateral da pirâmide é a soma das áreas de todas as faces laterais, portanto
Área lateral = $\,6 \centerdot 3\sqrt{427}\;=\;18\sqrt{427}\;$ que corresponde à alternativa
(A)
×
(ITA - 2004) Sejam as funções $\;f\;$ e $\;g\;$ definidas em $\;{\rm I\!R}\;$ por $\;f(x) = x^2 + \alpha x\; $ e $\;g(x) = -(x^2 + \beta x)\;$, em que $\alpha$ e $\beta$ são números reais. Considere que estas funções são tais que
$f$$g$
Valor
mínimo
Ponto de
mínimo
Valor
máximo
Ponto de
máximo
$-1$$< 0$$\frac{9}{4}$$> 0$

Então a soma de todos os valores de $\;x\;$ para os quais $\;(f \circ g)(x) = 0\;$ é igual a:
a)
0
b)
2
c)
4
d)
6
e)
8

 



resposta: (D)
×
"O mestre tachou-nos de indisciplinados."
Qual a função sintática da expressão grifada:

a) objeto direto
b) predicativo do sujeito
c) predicativo do objeto
d) complemento nominal
e) objeto indireto

 



resposta: C
×
Observe as duas frases seguintes:

I) O proprietário da farmácia saiu.
II) O proprietário saiu da farmácia.

Sobre elas são feitas as seguintes considerações:
Na I, da farmácia é adjunto adnominal.
Na II, da farmácia é adjunto adverbial.
Ambas as frases têm exatamente o mesmo significado.
Tanto em I como em II, da farmácia tem a mesma função sintática.
Destas quatro considerações:

a) apenas uma é verdadeira
b) apenas duas são verdadeiras
c) apenas três são verdadeiras
d) as quatro são verdadeiras
e) nenhuma é verdadeira

 



resposta: B
×
(MED. ABC) Sabendo que a oração subordinada substantiva apositiva exerce a função de aposto e que este "é um termo de natureza substantiva que se refere a outro, também de natureza substantiva" , marque a alternativa que apresenta uma oração apositiva:

a) Disse-me: vá embora.
b) Cometeu dois erros, aliás, três.
c) Havia apenas um meio de ajudá-la: contar-lhe a verdade.
d) "Como Sofia falasse das bonitas rosas que possuía, Rubião pediu para ir vê-las: era doido por flores."
e) Não preciso de ajuda: sei arrumar-me sozinho.

 



resposta: C
×
(FUND. LUSÍADAS) Em qual dos períodos há uma oração reduzida de infinitivo, substantiva, com a função objetiva indireta?
a)
Peço-te vires mais tarde.
b)
O que me admira é conservares tuas forças.
c)
Tua nota depende de saberes a lição.
d)
Cumpre-me puni-lo, infelizmente.
e)
Não sei se eles foram honestos.

 



resposta: Alternativa C
×
(U.F.VIÇOSA) "O médico sabia piano e tocava agradavelmente; a sua conversa era animada; sabia esses mil modos que entretêm geralmente as senhoras quando elas não gostam..."(M. de Assis)

A oração grifada no período desempenha a função sintática de:

a) predicado nominal
b) aposto
c) predicativo
d) complemento nominal
e) adjunto adnominal

 



resposta: E
×
(PUC) "... uma lagoa compreende as nossas pequeninas desventuras, o efêmero que nos fere."

Os termos "que" e "nos" exercem função sintática respectivamente:

a) conjunção integrante e objeto direto
b) pronome relativo (obj. dir.) e sujeito
c) pronome relativo (sujetito) e objeto direto
d) pronome relativo (sujeito) e objeto indireto
e) conjunção integrante e objeto indireto

 



resposta: C
×
Dê a expressão da altura de um triângulo equilátero em função da medida do lado do triângulo.

 



resposta: triângulo equilátero de lado l

Resolução:
No triângulo da figura:

$\;\ell^2 = h^2 + (\frac{\ell}{2})^2 \;\;\Longleftrightarrow \;\; h^2 = \ell ^2 - \frac{\ell ^2}{4} \; = \; \frac{3 \ell^2}{4}\;\Longrightarrow\;$

ou $\;\; h = \frac{\ell \sqrt{3}}{2}$


Resposta: $\;\;h = \frac{\ell \sqrt{3}}{2}$
×
Um prisma triangular regular tem a aresta da base igual à altura. Calcular a área total do sólido, sabendo-se que a área lateral é 10 m².

 



resposta:
prisma triangular regular

Considerações:

Se o prisma triangular é "regular" significa que as bases são triângulos equiláteros e as arestas laterais são perpendiculares aos planos que contém as bases ( → não é um prisma oblíquo).

$\phantom{XX}\,\left\{\begin{array}{rcr} a_{\large b} \longrightarrow & \\ h\;\longrightarrow\; & \\ A_{\mbox{base}} \longrightarrow & \\ \end{array} \right.\,$
aresta da base
altura do prisma$\; = a_{\large b}\,$
área da base, o triângulo equilátero
Resolução:
1. Sabemos que a área lateral é igual a $\;10 m^2\;$
A área lateral é a soma das áreas dos 3 retângulos que são as faces laterais do prisma (veja figura).
$\;A_{\mbox{lateral}} \;=\; 3 \centerdot a_{\large b} \centerdot h \;\;\Longrightarrow \;\; A_{\mbox{lateral}} \;=\; 3 (a_{\large b}) ^2\;\;$ então $\;\;\left(a_{\large b}\right)^2 \;=\; \dfrac{10}{3}$
2. Área da base:
(área do triângulo equilátero de lado $\;{\large \ell}\;$ em função da medida do lado do triângulo vale $\;\dfrac{\ell^2 \sqrt{3}}{4}\;$)
Então $\;A_{\mbox{base}} \;=\;\dfrac{\left(a_{\large b}\right)^2\sqrt{3}}{4}\;\;\Longrightarrow \;\;A_{\mbox{base}}\;=\dfrac{10}{3}\centerdot\dfrac{\sqrt{3}}{4}\;m^2\;\Longrightarrow$ $\; \;\;A_{\mbox{base}}\;=\dfrac{10\sqrt{3}}{12}\;m^2$
3. Área total:
$A_{\mbox{total}} \;=\;A_{\mbox{lateral}}\,+\,2\centerdot A_{\mbox{base}} \;\;\Longrightarrow \;\;A_{\mbox{total}}\;=\; 10\,+\,2 \centerdot \dfrac{10\sqrt{3}}{12}$
$\;\boxed{\;A_{total}\; = \;10(1 + \dfrac{\sqrt{3}}{6})\;m^2\;}\;$

×
Na figura, calcule "$\;x\;$" em função de $\;a\;$.
combinação de triângulos retângulos

 



resposta: Resolução:
$\;z^2\; = a^2 + a^2$
$\;y^2\; = z^2 + a^2 \; \Longrightarrow\; y^2 \; = a^2 + a^2 + a^2$
$\;w^2\; = y^2 + a^2\; \; \Longrightarrow\; w^2 = a^2 + a^2 + a^2 + a^2$
$\;x^2\; = w^2 + a^2 \;\Longrightarrow \; x^2 \; = 5 \centerdot a^2$
então
Resposta:
$\;x\; = \; a \sqrt{5}$
Observe que $\;x\; = a \centerdot \sqrt{n + 1}\;$, sendo $\;n\;$ o número de triângulos retângulos.
×
Calcular o lado $\;a\;$ de um triângulo $\;ABC\;$ sabendo-se que $\;\hat{B}\,=\,60^o\,\text{, } \hat{C}\,=\,45^o \;\text{ e }\; \overline{AB}\,=\, 2\text{ m}$.
triângulo ABC com 60 e 45 graus

 



resposta: Resolução:
$\,\triangle ADB \left\{ \operatorname{sen}60^o \,=\,{\large \frac{h}{2}}\; \Rightarrow \;h\,=\,\sqrt{3} \text{m.}\right.\,$
Então $\,BD^2 + (\sqrt{3})^2\,=\,2^2 \;\Rightarrow\;BD\,=\,1\text{m.}\,$
$\,\triangle ADC \left\{ \operatorname{tg}45^o \,=\,{\large \frac{\sqrt{3}}{CD}} \; \Rightarrow \; CD = \sqrt{3} \text{m.} \right.\,$
Logo: $\,a\,=\,BD\,+\,CD \;\Rightarrow\;$
$\boxed{\;a\,=\,(1\,+\,\sqrt{3})\text{ m.}\;}\,$
×
Sejam $\,A\,=\, \lbrace 5\rbrace \,$ e $\,B\,=\, \lbrace 3, 7 \rbrace \,$. Todas as Relações Binárias de $\,A\,$ em $\,B\,$ são:
a)
$\, \lbrace(5; 3) \rbrace \,$,$\, \lbrace (5; 7) \rbrace \,$ e $\, \lbrace(5; 3), (5; 7) \rbrace \,$
b)
$\,\varnothing\,$, $\, \lbrace (5; 3)\rbrace \,$, $\, \lbrace (5; 7)\rbrace \,$ e $\,A\,\times \,B$
c)
$\, \lbrace (5;3) \rbrace \;$ e $\; \lbrace (5; 7)\rbrace \,$
d)
$\,\varnothing\,$, $\,\lbrace \, (3; 5)\,\rbrace\,$, $\,\lbrace \, (7; 5)\,\rbrace\;$ e $\;A\,\times\,B$
e)
$\,\varnothing\,$, $\,\lbrace \, (5; 3)\,\rbrace\;$ e $\;\lbrace \, (5; 7)\,\rbrace\,$

 



resposta: (D)
×
Dados $\,A\,=\,\lbrace \, 2, 3, 4 \,\rbrace\,$ e $\, B\,=\,\lbrace \, 3, 4, 5, 6\,\rbrace\,$, seja $\,f\,$ a Relação Binária de $\,A\,$ em $\,B\,$ tal que $\,f\,=\,\lbrace \, (x; y)\,\in \,A \times B \,\mid x\;$divide$\; y \,\rbrace\,$
Então:
a)
$\,f\,=\,\lbrace \,(2;2), (3;3), (4;4) \,\rbrace\,$
b)
$\,f\,=\,\lbrace \, (3;3), (4;4)\,\rbrace\,$
c)
$\,f\,=\,\varnothing$
d)
$\,f\,=\,\lbrace \,(2;4), (2;6), (3;3), (3;6), (4;4) \,\rbrace\,$
e)
$\,f\,=\,\lbrace \, (4;2), (6;2),(3;3), (6;3), (4;4) \,\rbrace\,$

 



resposta: (D)
×
Se $\,n(A)\,=\,m\,$ e $\,n(B)\,=\,p\,$, então o número de Relações Binárias de $\,A\,$ em $\,B\,$, que não são vazias, é:
a)
$\,m \centerdot p$
b)
$\,m \centerdot p \, - \,1$
c)
$\,2^{m \centerdot p}\;$
d)
$\,2^{m \centerdot p} - 1$
e)
$\,2^{m \centerdot p - 1}$
 
 

 



resposta: (D)
×
(PUCC) São dados os conjuntos $\,A\,=\,\lbrace \, 3, 5, 6 \,\rbrace\,$ e $\,B\,=\,\lbrace \,4, 5, 9, 10, 12 \,\rbrace\,$ e a relação $\,R\,=\,\lbrace \,(x;y)\, \in \,A \times B\,\mid\,$ m.d.c$(x;y)\,=\,1 \,\rbrace\,$
O número de elementos da relação inversa de $\;R\;$ é:


a)8b)4c)10
d)6e)7



 



resposta: alternativa E
×
(PUC) O domínio da Relação $\,P\,=\,\lbrace \,(x;y)\,\in\,\mathbb{N}\times \mathbb{N}\,\mid \,y\,=\,x-5 \,\rbrace\,$ é:
a)
$\mathbb{N}$
b)
$\mathbb{N}^*$
c)
$\mathbb{R}$
d)
$\lbrace \,x\,\in\,\mathbb{N}\,\mid\,x \,\geqslant \,6\,\rbrace\,$
e)
$\lbrace \,x\,\in\,\mathbb{N}\,\mid\,x \,\geqslant \,5\,\rbrace\,$

 



resposta: (E)
×
(PUC) Dizemos que uma Relação entre dois conjuntos A e B é uma função de A em B quando — e apenas quando — todo elemento de
a)
B é imagem de algum elemento de A
b)
B é imagem de um único elemento de A
c)
A possui somente uma imagem em B
d)
A possui no mínimo uma imagem de B
e)
A possui somente uma imagem de B e vice-versa

 



resposta: (C)
×
(UFGO - 1982) No conjunto $\,R^2\,=\,\lbrace \,(x; y) \mid \, x,y\,\in\,\mathbb{R} \,\rbrace\,$ definimos:
1)$\,(x_1, y_1)\,=\,(x_2, \,y_2)\,\Longleftrightarrow\,x_1\,=\,x_2\;$ e $\;y_1\,=\,y_2$
2)$\,(x_1,\, y_1)\,+\,(x_2, \,y_2)\,=\,(x_1\,+\,x_2, y_1\,+\,y_2)$
3)$\,(x_1,\, y_1)\centerdot (x_2,\, y_2)\,=\,(x_1 x_2\,-\,y_1 y_2 \, ,\; x_1 y_2 \,+\, x_2 y_1)$

Com base nas definições, resolver a equação:
$(x,\, y)\centerdot(1, \,2) \, + \, (2,\, 3)\,=\,(4, \, 5)$

 



resposta: $\,x\,=\,\frac{6}{5}\,$ e $\,y\,=\,- \frac{2}{5}$ ou $(\frac{6}{5};-\frac{2}{5})$
×
Se $\,A\,$ é um conjunto tal que $\,n(A \times A)\,=\,9\;$ e que $\,\lbrace \, (2;4), (4;5)\,\rbrace\, \subset \,A \times A\,$, determinar $\,A \times A\,$.

 



resposta: $\,A \times A\,=\,\lbrace \, (2;2),(2;4),(2;5),(4;2),(4;4),(4;5),(5;2),(5;4),(5;5) \,\rbrace\,$
×
(PUCC) Sejam $\,M\,=\,\lbrace \,x\in \mathbb{R}\;\mid\; 0 \, \leqslant \, x \, \leqslant 5 \,\rbrace\,$ e $\,P\,=\,\lbrace \,x\in \mathbb{R}\;\mid\; 3 \, \leqslant \, x \, \leqslant 7 \,\rbrace\,$. O conjunto $\,(M\,-\,P)\,\times\,(P\,-\,M)\,$ é representado pela região:
produto cartesiano m menos p por p meno m
a)
$\,R_1\phantom{XX}$
b)
$\,R_2$
c)
$\,R_3\phantom{XX}$
d)
$\,R_4\,$
e)
$\,R_1 \, \cup \, R_4\,$
 
 

 



resposta: (D)
×
(PUCC - 1982) Dados os conjuntos $A\,=\,\lbrace \,3,\, 4,\, 6 \,\rbrace\,$, $\;B\,=\,\lbrace \,1,\, 2\,\rbrace\,$ e $\,C\,=\,\lbrace \,3,\, 6,\, 9,\,12 \,\rbrace\,$, determine o conjunto $\,(C\,-\,A)\, \times\,B\,$.


 



resposta: $\,(C\,-\,A)\, \times\,B\; = \,\lbrace \, 9,12\,\rbrace\,\times \,\lbrace \, 1,2\,\rbrace\;=\;\,\lbrace \, (9;1),(9;2),(12;1),(12;2)\,\rbrace\,$

×
(PUCC - 1982) Dados os conjuntos $\;A\,=\,\lbrace \,x\,\in\,\mathbb{R}\;\mid\;1\,\leqslant\,x\,\leqslant\,3 \,\rbrace\;$ e $\;B\,=\,\lbrace \,x\,\in\,\mathbb{R}\;\mid\;-1\,\leqslant\,x\,\leqslant\,1\,\rbrace\;$ represente, graficamente, o produto cartesiano $\,B\, \times\,A\,$.

 



resposta: produto cartesiano de A por B
×
O gráfico representa uma relação binária de $\,A\,$ em $\,B\,$. Responda em relação ao gráfico:
a)
Se representa ou não uma função de $\,A\,$ em $\,B\,$;
b)
em caso afirmativo, determinar o DOMÍNIO, o CONTRADOMÍNIO e o CONJUNTO IMAGEM da mesma.
função de A em B

 



resposta: a) é função.
b) D(f) = [1;4]
CD(f) = [1;3]
Im(f) = [2;3]

×
Em relação ao gráfico a seguir que representa uma relação binária de $\,A\,$ em $\,B\,$, responda as questões:
a)
Se o gráfico representa ou não uma função de $\,A\,$ em $\,B\,$;
b)
Em caso afirmativo, determinar o DOMÍNIO, o CONTRADOMÍNIO e o CONJUNTO IMAGEM da mesma.
gráfico cartesiano de uma relação binária entre conjuntos

 



resposta: não é uma função.

×
Em relação ao gráfico a seguir que representa uma relação binária de $\,A\,$ em $\,B\,$, responda as questões:Se o gráfico representa ou não uma função de $\,A\,$ em $\,B\,$;Em caso afirmativo, determinar o DOMÍNIO, o CONTRADOMÍNIO e o CONJUNTO IMAGEM da mesma.
relação binária entre os conjuntos A e B

 



resposta: a) é função
b) D(f) = [1;4]
CD(f) = [1;3]
$\,Im(f)\,=\,$ $\lbrace y \in \mathbb{R} \;\mid\; 1 \leqslant y < 2\,$ ou $\, y = 3 \rbrace$
×
(MED JUNDIAÍ - 1982) O domínio da função $\,f\;$, definida por $\,f(x)\,=\, \frac{\sqrt{2x - 1}}{2x - 1}\,$, é:
a)
$\,\lbrace \, x\; \mid \; x \in \mathbb{R}\;$ e $\;x \neq \frac{1}{2} \,\rbrace\,$
b)
$\,\lbrace \, x\; \mid \; x \in \mathbb{R}\;$ e $\;x > \frac{1}{2} \,\rbrace\,$
c)
$\,\lbrace \, x\; \mid \; x \in \mathbb{R}\;$ e $\;x \geqslant \frac{1}{2} \,\rbrace\,$
d)
$\, \mathbb{R} _+$
e)
$\, \mathbb{R}$

 



resposta: (B)
×
(FMU) Se $\,f(x)\,=\, 2x^3\,-\,1\,$, então $\phantom{X}f(0)\,+\,f(-1)\,+\,f(\frac{1}{2})\phantom{X}$ é igual a:
a)
$\,\frac{-3}{4}\,$
b)
$\,\frac{-15}{4}\,$
c)
$\,\frac{-19}{4}\,$
d)
$\,\frac{-17}{4}\,$
e)
$\,\frac{-13}{4}\,$

 



resposta: (C)
×
(OSEC) Seja $\,f\,$ a função tal que
$\,f(x)\,=$ $\,x^3\,-\,8\,+\,(x^2\,+\,2x\,+\,4) \centerdot (2\,-\,x)\,$
O conjunto de todas as soluções da equação $\,f(x)\,=\,0\,$ é:
a)
$\;\varnothing\phantom{XX}$
b)
$\,\lbrace \, 2\,\rbrace\,$
c)
$\,\lbrace \, -2\,\rbrace\,$
d)
$\,\lbrace \, -2,\,2\,\rbrace\,$
e)
$\,{\rm I\!R}\,$
 
 

 



resposta: (E)
×
Calcule $\, f(2)\,$, sabendo-se que $\;f(x-1) \, = \, x^2\, ,\; \vee \negthickspace \negthickspace \negthickspace \negthinspace - \, x \in \mathbb{R} \,$.

 



resposta: $f(3 - 1) = 3^2\; \Longrightarrow\;$ f(2) = 9

×
(PUC) Dada a função $f(x)\,=\,\left\{ \begin{array}{rcr} 3^{-x} - 1\; & \mbox{, se } -1 \leqslant x < 0 \\ tg(\frac{x}{2})\; & \mbox{, se } \; 0 \leqslant x < \pi \phantom{X}\\ {\large \frac{x}{x^2 - 2}}\; & \mbox{, se } \; \pi \leqslant x \leqslant 6\phantom{X} \\ \end{array}\right. \;\;$

então $\,f(-1)\,$,$\,f(\frac{\pi}{2})\,$ e$\,f(4)\,$ são, respectivamente

a)
$2, \, 1, \,{\large \frac{2}{7}}$
b)
$1, \, 3, \, {\large \frac{2}{3}}$
c)
$2, \, 3, \, {\large \frac{1}{2}}$
d)
$3, \, 2, \,{\large \frac{-1}{2}}$
e)
${\large \frac{1}{2}}, \, 1, \, 2 $

 



resposta: (A)
×
(FUVEST) As funções $\,f\,$ e $\,g\,$ são dadas por:
$\left \{ \begin{array} {rcr} f(x)\,=\,{\large \frac{3}{5}}x - 1 \\ g(x)\,=\,{\large \frac{4}{3}}x + a \\ \end{array}\right.$
Sabe-se que $\;f(0)\,-\,g(0) = {\large \frac{1}{3}}\,$. O valor de $\;f(3)\,-\,3 \centerdot g({\large \frac{1}{5}})\;$ é:
a)
0
b)
1
c)
2
d)
3
e)
4

 



resposta: (E)
×
(FEI) Seja $\,f\,:\, \mathbb{R} \rightarrow \mathbb{R}\,$ a função tal que:
$\,f(x)\,=\, x^2\,+\,bx\,+\,c\,$
Calcule $\,b \centerdot c\;$ sabendo-se que $\,f(-1)\,=\, 1\;$ e $\,f(1)\,=\, \alpha\,$.

 



resposta: $ b \centerdot c \,=\, (\frac{\alpha - 1}{2})^2\,$

×
(UEMT) Para que os pontos (1; 3) e (3, -1) pertençam ao gráfico da função dada por $\,f(x)\,=\, ax + b\,$, o valor de $\,b - a\,$ deve ser
a)
7
b)
5
c)
3
d)
-3
e)
-7

 



resposta: (A)
×
(MACKENZIE) A função $\,f\,:\, \mathbb{R} \rightarrow \mathbb{R}\,$ é tal que
$\,f(3x)\,=\,3\centerdot f(x)\,$, $\, \vee \negthickspace \negthickspace \negthickspace \negthinspace - \, x \,\in\,\mathbb{R}\,$
Se $\,f(9)\,=\,45\,$, então $\,f(1)\,+\,f(3)\,$ é igual a:
a)
15
b)
5
c)
20
d)
10
e)
25


 



resposta: alternativa C
×
(CESGRANRIO) A função $\,f\,$ satisfaz a relação:
$\,f(x+1)\,=\,x \centerdot f(x)\,$, $\, \vee \negthickspace \negthickspace \negthickspace \negthinspace - x \,\in\, \, \mathbb{R^{\large *}} \negthickspace \negthinspace {_+} $
Se $\,f({\large \frac{1}{2}}) \,=\, \sqrt{{\large \pi}}\;$, calcule $\;f({\large \frac{3}{2}})$

 



resposta: $\,{\large \frac{\sqrt{\pi}}{2}}\,$
×
(FAAP) Dada a função $\,f(x)\,=\, 2x^2 \, + \, 1\,$, se $\,\Delta f \,=\, f(x)\,-\,f(3)\,$, expressar $\,\Delta f\,$ somente em termos de $\,\Delta x\,$, sendo $\,\Delta x \,=\, x\,-\,3\,$.

 



resposta: $\,2 \Delta x (\Delta x \,+\,6)\,$

×
(FMU) O domínio da função $\,f(x)\,=\, \dfrac{\sqrt{4x\,-\,x}}{x}\,$ é:
a)
$\,[4;\,+\infty[\,$
b)
$\,]-\infty;\,4]\,$
c)
$\,\lbrace \, x \in \mathbb{R} \; \mid \; x \leqslant 4 \;\;\mbox{e}\;\; x \, \neq \, 0 \,\rbrace\,$
d)
$\,\mathbb{R}\,-\,\lbrace 4 \rbrace\,$
e)
$\,\mathbb{R}^{\large{*}}\,$

 



resposta: (B)
×
(UEMT) O domínio e o contradomínio de uma função $\,f\,$ são subconjuntos de $\,\mathbb{R}\,$. Sendo $\,f\,$ dada por $\,f(x)\,=\, {\large \dfrac{1}{\sqrt{x - x^2}}}\,$ o dominio de $\,f\,$ pode ser:
a)
[0; 1]
b)
[0; 1[
c)
]0; 1[
d)
]1;$\,+\infty\,$[
e)
]$\,-\infty\,$; 0[

 



resposta: (C)
×
(MACKENZIE) Se $\,f\,$ é tal que $\,f(x\,+\,1) = {\dfrac{\;3x\,+\,5\;}{\;2x\,+\,1\;}},\,x\,\neq\,\dfrac{\;-1\;}{\;2\;}\,$, então o domínio de $\,f\,$ é:
a)
$\,\mathbb{R}\,-\,\lbrace \, \frac{1}{2}\,\rbrace\phantom{X}$
b)
$\,\mathbb{R}\,-\,\lbrace \, \frac{-1}{2}\,\rbrace\,$
c)
$\,\mathbb{R}\,-\,\lbrace \, \frac{-5}{3}\,\rbrace\,$
d)
$\,\mathbb{R}\,-\,\lbrace \, \frac{5}{3}\,\rbrace\,$
e)
$\,\mathbb{R}\,-\,\lbrace \, \frac{-3}{5}\,\rbrace\,$

 



resposta: (A)
×
(MAUÁ) Seja $\,f\,:\, \mathbb{R} \rightarrow \mathbb{R}\,$ a função tal que $\,f(x)\,=\,x^2\,$. Seja $\,g\,:\, \mathbb{R} \rightarrow \mathbb{R}\,$ a função tal que $\,g(x)\,=\,{\large \frac{f(x \,+\, h) \,- \,f(x)}{h} }\,$. Assim, $\,g(x)\,$ é igual a:
a)
h
b)
x
c)
2x
d)
2x + h
e)
x + h

 



resposta: (D)
×
(ITA) Seja $\,f\,:\, \mathbb{R} \rightarrow \mathbb{R}\,$ a função definida por $\,f(x)\,=\,ax\,+\,b\,$, onde $\,a \in \mathbb{R}^{\large *} \,$ e $\,b \in \mathbb{R}\,$.
Se $\,\alpha \, \in \mathbb{R}\,$, $\,\beta \, \in \mathbb{R}\,$ e $\,\alpha \neq \beta \,$, demonstre que $\,{\large\frac{f(\alpha) \, - \, f(\beta)}{\alpha \,-\, \beta}}\,=\, a\,$


 



resposta: Resolução:
$\,f(x)\,=\,ax\,+\,b\,$
$\,f(\alpha)\,=\,a\alpha\,+\,b\,$
$\,f(\beta)\,=\,a\beta\,+\,b\,$
$\,{\large\frac{f(\alpha) \, - \, f(\beta)}{\alpha \,-\, \beta}}\,=\, $
$=\, {\large \frac{a \alpha \,+\,b\,-\,(a\beta\,+\,b)}{(\alpha \,-\, \beta)}}\,=$
$=\,{\large \frac{a\alpha \,+\,b \, - a \beta \, - \, b}{\alpha \, - \, \beta}}\,=$
$=\,{\large \frac{a\alpha \, - \, a\beta}{\alpha \,-\, \beta}}\,$=
$=\,{\large \frac{a(\alpha \,-\, \beta)}{\alpha \,-\, \beta}} \,=\, a\,$

×
(PUC) Seja $\,D\,=\,\lbrace \, 1, \,2, \,3, \,4, \,5 \,\rbrace\,$, e $\,f\,:\, D \rightarrow \mathbb{R}\;$ a função definida por $\,f(x)\,=\,(x\,-\,2)\centerdot(x\,-\,4)\,$. Então:
a)
$f\,$ é sobrejetora
b)
$f\,$ é injetora
c)
$f\,$ é bijetora
d)
O conjunto imagem de $\,f\,$ possui 3 elementos somente
e)
$\,Im(f)\,=\,\lbrace \, -1,\,0,\,1 \,\rbrace\,$

 



resposta: (D)
×
(STA MARIA - MANAUS) O número de funções injetoras definidas em $\,A\,=\,\lbrace \,1, 2 \,\rbrace\,$ com valores em $\,B\,=\,\lbrace \,0,\,1,\,2,\,3\,\rbrace\;$ é:
a)
10
b)
12
c)
60
d)
125
e)
243

 



resposta: (B)
×
(UBERLÂNDIA) Qual a afirmativa CERTA?
a)
Se $\,f\,:\, \mathbb{R} \rightarrow \mathbb{R}\,$ tal que $\,f(x)\,=\,c\,\neq\,0\,$ para todo $\,x\,\in\,\mathbb{R}\,$, então $\,f(2x)\,=\,2c\,$.
b)
Se $\,f(x)\,=\,5^x\,$, então $\,f(y\,+\,z)\,=\,f(y) \centerdot f(z)\,$ para todo $\,z\,$ e $\,y\,$ reais.
c)
Toda função constante é também função ímpar.
d)
Se $\,f(x)\,=\,{\large (\frac{1}{2})^x }\;$ e $\; x\,<\,0\,$, então $\,f(x)\, < \,0\,$.
e)
Se $\,f\,:\, \mathbb{R} \rightarrow \mathbb{R}\;$ e $\;f(x)\,+\,f(-x)\,=\,0\,$, então $\,f\,$ é uma função par.

 



resposta: (B)
×
(USP) Dizemos que uma função real é par se $\,f(x)\,=\,f(-x)\,$ e que é ímpar se $\,f(x)\,=\,-f(-x)\,$.
Das afirmativas que seguem indique qual a falsa:
a)
O produto de duas funções ímpares é uma função ímpar.
b)
O produto de duas funções pares é uma função par.
c)
A soma de duas funções ímpares é uma função ímpar.
d)
A soma de duas funções pares é uma função par.
e)
Alguma das afirmações anteriores é falsa.

 



resposta: alternativa A (é falsa)
×
(ITA) Com relação à função $\,f\,:\, \mathbb{R^{\large *}} \negthickspace \negthinspace {_+} \rightarrow \mathbb{R}_+\;$ tal que $\,f(b\,-\,a)\,=\,f(a)\,-\,f(b);\,\vee \negthickspace \negthickspace \negthickspace \negthinspace - a,b \,\in\, \, \mathbb{R^{\large *}} \negthickspace \negthinspace {_+} \,$, então:
a)
$\,f\,$ é decrescente em $\,\mathbb{R^{\large *}} \negthickspace \negthinspace {_+}\,$
b)
$\,f\,$ é crescente em $\,\mathbb{R^{\large *}} \negthickspace \negthinspace {_+}\,$
c)
$\,f\,$ é estritamente decrescente em $\,\mathbb{R^{\large *}} \negthickspace \negthinspace {_+}\,$
d)
$\,f\,$ é estritamente crescente em $\,\mathbb{R^{\large *}} \negthickspace \negthinspace {_+}\,$
e)
$\,f\,$ é constante em $\,\mathbb{R^{\large *}} \negthickspace \negthinspace {_+}\,$

 



resposta: (A)
×
(PUC) Qual das funções abaixo é função par?
a)
$\,f(x)\,=\,{\large \frac{1}{x^2}}\,$
b)
$\,f(x)\,=\,{\large \frac{1}{x}}\,$
c)
$\,f(x)\,=\,x\phantom{X}$
d)
$\,f(x)\,=\,{\large x^5}\,$
e)
$\,f(x)\,=\, \operatorname{sen}(x)\,$
 
 

 



resposta: (A)
×
(PUC) Uma função que verifica a propriedade
"qualquer que seja $\,x\,$, $\;f(-x)\,=\,-f(x)\,$" é:
a)
$f(x) \,=\, 2\phantom{X}$
b)
$f(x)\, =\, 2x$
c)
$f(x)\,=\,x^2\;$
d)
$\,f(x)\,=\,2^x\,$
e)
$\,f(x)\,=\, \operatorname{cos}(x)\,$
 
 

 



resposta: (B)
×
Sejam $\,f\,:\, \mathbb{R} \rightarrow \mathbb{R}\;$, $\,g\,:\, \mathbb{R} \rightarrow \mathbb{R}\;$ e $\,h\,:\, \mathbb{R} \rightarrow \mathbb{R}\;$ três funções definidas por $\,f(x)\,=\,x\,+\,1\;$,$\,g(x)\,=\,x^2\,\,+x\,+\,1\;$ e $\,h(x)\,=\,1\,-\,x\;$. Determine $\,g \circ f\;$, $\;f \circ g\;$, $\;h \circ f\;$, $\;f \circ h\;$,$\;h \circ g\;\,$ e $\;\,g \circ h\,$.

 



resposta: Resolução:
a) $\,(g \circ f)(x)\,=\,g \left[f(x)\right]\,=\,\left(f(x)\right)^2\,+\,f(x)\,+1\,=$
$\,=\,(x + 1)^2\,+\,(x+1)\,+\,1\,=\,(x^2\,+\,2x\,+\,1)\,+\,(x+1)\,+\,1\,=$
$\,=\,x^2\,+\,3x\,+\,3\,$
Logo $ \left\{\begin{array}{rcr} &g \circ f : \, \mathbb{R} \rightarrow \mathbb{R}\; \phantom{XXXXX} \\ &(g \circ f)(x)\,=\,x^2\,+\,3x\,+\,3 \\ \end{array} \right.$
b) $\,(f \circ g)(x)\,=\,f \left[g(x)\right]\,=\,g(x)\,+\,1\,=\,(x^2\,+x\,+\,1)\,+\,1\,=\,x^2\,+\,x\,+\,2$
Logo $ \left\{\begin{array}{rcr} &f \circ g : \, \mathbb{R} \rightarrow \mathbb{R}\; \phantom{XXXXX} \\ &(f \circ g)(x)\,=\,x^2\,+\,2x\,+\,2 \\ \end{array} \right.$
c) $\,(h \circ f)(x)\,=\,h \left[f(x)\right]\,=\,1 \,-f(x)\,=\,1\,-\,(x\,+\,1)\,=\,1\,-\,x\,-1\,=\,-x$
Logo $ \left\{\begin{array}{rcr} &h \circ f : \, \mathbb{R} \rightarrow \mathbb{R}\; \phantom{XXXXX} \\ &(h \circ f)(x)\,=\,-x \\ \end{array} \right.$
d) $\,(f \circ h)(x)\,=\,f \left[h(x)\right]\,=\,h(x)\,+\,1\,=\,(1\,-\,x)\,+\,1\,=\,2\,-\,x$
Logo $ \left\{\begin{array}{rcr} &f \circ h : \, \mathbb{R} \rightarrow \mathbb{R}\; \phantom{XXXXX} \\ &(f \circ h)(x)\,=\,2\,-x \\ \end{array} \right.$
e) $\,(h \circ g)(x)\,=\,h \left[g(x)\right]\,=\,1 \,-g(x)\,=\,1\,-\,(x^2\,+\,x\,+1)\,=$
$\,=\,1\,-\,x^2\,-x\,-1\,=\,-x^2 - x$
Logo $ \left\{\begin{array}{rcr} &h \circ g : \, \mathbb{R} \rightarrow \mathbb{R}\; \phantom{XXXXX} \\ &(h \circ g)(x)\,=\,-x^2\,-\,x \\ \end{array} \right.$
f) $\,(g \circ h)(x)\,=\,g \left[h(x)\right]\,=\,\left(h(x)\right)^2\,+\,h(x)\,+\,1\,=$
$\,=\,(1\,-\,x)^2 \,+\,(1\,-\,x)\,+\,1\,=\,(1\,-\,2x\,+\,x^2)\,+\,(1\,-\,x)\,+\,1\,=$
$\,=\,x^2\,-\,3x\,+\,3$
Logo $ \left\{\begin{array}{rcr} &g \circ h : \, \mathbb{R} \rightarrow \mathbb{R}\; \phantom{XXXXX} \\ &(g \circ h)(x)\,=\,x^2\,-\,3x\,+\,3 \\ \end{array} \right.$
É muito importante notar que $\; \left\{\begin{array}{rcr} g \circ f & \neq & f \circ g \\ h \circ f & \neq & f \circ h \\ h \circ g & \neq &g \circ h \\ \end{array} \right.$

×
Seja $\,f\,:\, \mathbb{R} \rightarrow \mathbb{R}\;$ a função definida por $\,f(x)\,=\,x\,+\,1\;$. Determine $\;\,f \circ f\;\,$, $\;\;f \circ f\, \circ f\;\,$ e $\;\;f \circ f\, \circ f\, \circ f\;$.

 



resposta: Resolução:
a) $\,(f \circ f)(x)\,=\,f \left[f(x)\right]\,=\, f(x)\,+\,1\,=\,(x\,+\,1)\,+\,1=\,x\,+\,2$
Portanto: $ \left\{\begin{array}{rcr} & f \circ f : \, \mathbb{R} \rightarrow \mathbb{R}\; \phantom{XX} \\ &(f \circ f)(x)\,=\,x\,+\,2 \\ \end{array} \right.$
b) $\,(f \circ f \circ f)(x)\,=\,(f \circ f) \left[f(x)\right]\,=\,f(x)\,+\,2\,=\,(x\,+\,1)\,+\,2\,=\,x\,+\,3$
Portanto: $ \left\{\begin{array}{rcr} &f \circ f \circ f : \, \mathbb{R} \rightarrow \mathbb{R}\; \phantom{XX} \\ &(f \circ f \circ f)(x)\,=\,x\,+\,3 \\ \end{array} \right.$
c) $\,(f \circ f \circ f \circ f)(x)\,=\,(f \circ f \circ f) \left[f(x)\right]\,=\,f(x)\,+\,3\,=\,(x\,+\,1)\,+\,3\,=$
$\,=\,x\,+\,4\,$
Portanto $ \left\{\begin{array}{rcr} &f \circ f \circ f \circ f : \, \mathbb{R} \rightarrow \mathbb{R}\; \phantom{XX} \\ &(f \circ f \circ f \circ f)(x)\,=\,x\,+\,4 \\ \end{array} \right.$



×
Sejam $\,f\;$ e $\,g\;$ duas funções de $\,\mathbb{R} \rightarrow \mathbb{R}\,$ definidas por:

$\;f(x)\,=\; \left\{\begin{array}{rcr} x\,+\,3\; \mbox{, se}& x \leqslant 3 \\ x\,-\,4\; \mbox{, se} & x \geqslant 3 \\ \end{array} \right.$

$\,g(x)\,=\,2x\,-\,7\,$,$\;\;\vee \negthickspace \negthickspace \negthickspace \negthinspace - x \,\in\, \, \mathbb{R}$

Determine $\;f \circ g\;$ e $\,g \circ f\;$.

 



resposta: Resolução:

a) $\,(f \circ g)(x)\,=\,f \left[g(x)\right]\,=\,\left\{\begin{array}{rcr} g(x)\,+\,3 & \mbox{, se}\;\; g(x) \leqslant 3 \\ g(x)\,-\,4 & \mbox{, se}\;\; g(x) > 3 \\ \end{array} \right. \; \Longrightarrow$
$\,\Longrightarrow (f \circ g)(x)\,=\, \left\{\begin{array}{rcr} (2x\,-\,7)\,+\,3 & \mbox{, se}\;\; g(x) \leqslant 3 \phantom{XX} \\ (2x\,-\,7)\,-\,4 & \mbox{, se}\;\; (2x\,-\,7) > 3 \\ \end{array} \right. \; \Longrightarrow $
$\,\Longrightarrow (f \circ g)(x)\,=\, \left\{\begin{array}{rcr} 2x\,-\,4 \phantom{X} & \mbox{, se}\;\; x \leqslant 5 \\ 2x\,-\,11 \phantom{X} & \mbox{, se}\;\; x > 5 \\ \end{array} \right. \;$
b) $\,(g \circ f)(x)\,=\,g \left[f(x)\right]\,=\, \,2 \centerdot f(x) \,-\,7\,= \left\{\begin{array}{rcr} 2 \centerdot (x\,+\,3)\, - \,7 & \mbox{, se}\;\; x \leqslant 3 \\ 2 \centerdot (x\,-\,4) \,-\, 7 & \mbox{, se}\;\; x > 3 \\ \end{array} \right. \; \Longrightarrow$
$\,\Longrightarrow (g \circ f)(x)\,=\; \left\{\begin{array}{rcr} 2x\,-\,1 \; & \mbox{, se}\;\; x \leqslant 3 \\ 2x\,-\,15 & \mbox{, se}\;\; x > 3 \\ \end{array} \right. \; $
Portanto:
$\,f \circ g \; : \,\mathbb{R} \rightarrow \mathbb{R}\,$
$(f \circ g)(x)\,=\, \left\{\begin{array}{rcr} 2x\,-\,4 \; & \mbox{, se}\;\; x \leqslant 5 \\ 2x\,-\,11 & \mbox{, se}\;\; x > 5 \\ \end{array} \right. \;$
$\,g \circ f \; : \,\mathbb{R} \rightarrow \mathbb{R}\,$
$(g \circ f)(x)\,=\; \left\{\begin{array}{rcr} 2x\,-\,1 \; & \mbox{, se}\;\; x \leqslant 3 \\ 2x\,-\,15 & \mbox{, se}\;\; x > 3 \\ \end{array} \right. \; $



×
Seja $\,f\,:\, \mathbb{R}_+ \rightarrow \mathbb{R}_+\;$ a função definida por $\,f(x)\,=\,x^2\;$.
Determine uma função $\,g\,:\, \mathbb{R}_+ \rightarrow \mathbb{R}_+\;$ tal que a função composta $\;(f \circ g)\;$ seja uma
função identidade.

 



resposta: Resolução:
De $\;(f \circ g)\,=\,id\;$ decorre que:
$\,(f \circ g)(x)\,=\,id(x) \, \mbox{, } \; \vee \negthickspace \negthickspace \negthickspace \negthinspace - x \,\in\, \, \mathbb{R}_+ \, \Rightarrow $
$\Rightarrow\,f \left[g(x)\right]\,=\,x\, \mbox{, }\vee \negthickspace \negthickspace \negthickspace \negthinspace - x \,\in\, \, \mathbb{R}\,\Rightarrow $
$\Rightarrow \left[g(x)\right]^2 \,=\,x\,\mbox{, }\;\vee \negthickspace \negthickspace \negthickspace \negthinspace - x \,\in\, \, \mathbb{R}_+ \,$(pois $\,f(x)\,=\,x^2\,\mbox{, } \vee \negthickspace \negthickspace \negthickspace \negthinspace - x \,\in\, \, \mathbb{R}_+$)$\;\Rightarrow$
$\,\Rightarrow\,g(x)\,=\,+ \sqrt{x}\,$, (pois $\,g(x) \geqslant 0$).
Portanto:
$\,\left\{\begin{array}{rcr} \, & g\,:\, \mathbb{R}_+ \rightarrow \mathbb{R}_+\; \\ \, & g(x)\,=\,+\,\sqrt{x} \\ \end{array} \right.\,$


×
(UBERLÂNDIA) Qual das seguintes funções representa uma função injetora com dominio em A e imagens em B:
a)
relacao entre dois conjuntos a e b diagrama de venn
b)
relação entre conjuntos a e b diagrama de venn com flechinhas
c)
diagrama de função com flechinhas
d)
diagrama de Venn-Euler representando uma função de A em B com flechinhas
e)
função de A e B em diagrama de Venn-Euler

 



resposta: (E)
×
(UBERABA) Dentre os gráficos abaixo, o que melhor se adapta a uma função bijetora (injetora e sobrejetora) com domínio $\,\mathbb{R}\,$ e contradomínio $\,\mathbb{R}\,$ é:
a)
relação binária cobrinha
b)
relação binária subida
c)
relação binária bumerangue
d)
função bijetora
e)
função R em R

 



resposta: (D)
×
(PUC - BA) O gráfico seguinte é da função $\,f(x)\,$.
gráfico de efe de x
A sentença verdadeira é:
a)
$f(1)\,=\,1\,$;
b)
o domínio de $\;f(x)\;$ é $\,\lbrace\,x\in\mathbb{R}\;\mid\;x\neq 0 \,\rbrace\,$;
c)
o conjunto imagem de $\;f(x)\;$ é $\,\lbrace\,y\in\mathbb{R}\;\mid\;y > 0 \,\rbrace\,$;
d)
$f(x)\,$ é decrescente para $\,0 < x < 1\,$;
e)
$\,f(x)\,=\,\mid x\, \mid \,$, para $\,x < 0\,$ ou $\,x > 1\,$.

 



resposta: (D)
×
(ITA) Supondo $\,a < b\;$, onde $\;a\;$ e $\;b\;$ são constantes reais, considere a função
$\,H(x)\,=\,(b\,-\,a)x\,+\,a\,$
definida em $\,[0; 1]\,$. Podemos assegurar que:
a)
$\,H\,$ não é uma função injetora.
b)
Dado $\,y_0 < b\,$, sempre existe $\,x_0\,$ em $\,[0; 1]\,$, tal que $\,H({\large x_0})\,=\,y_0\,$
c)
Para cada $\,y_0\,$, com $\,a < y_0 < b\,$, corresponde um único $\,x_0\,$ em $\,[0; 1]\,$ tal que $\,H({\large x_0})\,=\,y_0\,$
d)
Não existe uma função real $\,G\,$, definida em $\,[a; b]\,$ tal que $\;(G \circ H)(x)\,=\,x\;$ para cada $\,x\,$ em $\,[0; 1]\,$
e)
$\,H\,:\,[0; 1] \rightarrow [a; b]\,$ não é sobrejetora.

 



resposta: (C)
×
(FUVEST) Se $\;f\,:\, {\rm I\!R}\; \rightarrow \; {\rm I\!R} \;$ é da forma $\,f(x)\,=\,ax\,+\,b\;$ e verifica $\,(f \circ f)(x)\,=\,x\,+\,1\;$, para todo $\,x\,$ real, então $\,a\,$ e $\,b\,$ valem, respectivamente:
a)
$\,1 \mbox{ e } \frac{1}{2}\phantom{XX}$
b)
$\,-1 \mbox{ e } \frac{1}{2}\,$
c)
1 e 2 
d)
1 e -2
e)
-1 e qualquer
 
 

 



resposta: (B)
×
(UBERLÂNDIA) Se $\,f(x\,-\,2)\,=\,2x^2\;, \vee \negthickspace \negthickspace \negthickspace \negthinspace - x \,\in\, \, \mathbb{R}\,$, então $\,f(x\,+\,2)\,$ é igual a:
a)
$\,2(x\,+\,2)^2\,$
b)
$\,2(x\,-\,2)^2\,$
c)
$\,2(2x)^2\phantom{XX}$
d)
zero
e)
$\,2(x\,+\,4)^2\,$

 



resposta: (E)
×
(FMU - FIAM) O valor de $\,\operatorname{sen}x \,+\,{\large \frac{\operatorname{sen}^3 x}{2}} \,+ \, {\large \frac{\operatorname{sen}^5 x}{4}} \,+\,...\;$ é:
a)
$\,\dfrac{\operatorname{sen}x}{1\,+\,\operatorname{sen^2}x}\,$
b)
$\,\dfrac{\operatorname{cos}x}{1\,-\,\operatorname{sen^2}x}\,$
c)
$\,\dfrac{\operatorname{sen}x}{1\,+\,\operatorname{cos^2}x}\,$
d)
$\,\dfrac{\operatorname{sen}x}{1\,-\,\operatorname{sen^2}x}\,$
e)
$\,\dfrac{2\operatorname{sen}x}{1\,+\,\operatorname{cos^2}x}\,$

 



resposta: (E)
×
(VUNESP) Se $\;x \,\text{, }\;y\;$ são números reais tais que:
$\,y\,=\, \dfrac{ \operatorname{cos^3}x \,-\, 2 \, \centerdot \,\operatorname{cos}x \,+\, \operatorname{sec}x }{ \operatorname{cos}x \, \centerdot \,\operatorname{sen^2}x } \;$, então:
a)
$\,y\,=\,\operatorname{sec^2}x\,$
b)
$\,y\,=\,\operatorname{tg^2}x\,$
c)
$\,y\,=\,\operatorname{cos^2}x\,$
d)
$\,y\,=\,\operatorname{cossec^2}x\,$
e)
$\,y\,=\,\operatorname{sen^2}x\,$

 



resposta: (B)
×
(VUNESP) Sejam $\;A\;$, $B$ e $C \;$ conjuntos de números reais. Sejam $\;f\,:\, A \rightarrow B \;$ e $\;g\,:\, B \rightarrow C \;$ definidas, respectivamente, por:
$\left\{\begin{array}{rcr} \,f(x)\, &=\,\operatorname{sen}x \text { , } \vee \negthickspace \negthickspace \negthickspace \negthinspace - x \text{ , }\,x \,\in\, \, A\,\; \phantom{XX} \\ \,g(x)\,&=\,{\Large \frac{1}{1\,-\,x^2}} - 1 \text{ , }\vee \negthickspace \negthickspace \negthickspace \negthinspace - x \text{ , }\,x \,\in\, \, B \\ \end{array} \right.\,$
Se existe $\;f\,:\, A \rightarrow C \;$, definida por $\,h(x)\,=\,g{\large [f(x)]} \text{, }\vee \negthickspace \negthickspace \negthickspace \negthinspace - x \text{, }\,x \,\in\, \, A\;$, então:
a)
$\,h(x)\,=\,\operatorname{cos}x\,$
b)
$\,h(x)\,=\,\operatorname{cos^2}x\,$
c)
$\,h(x)\,=\,\operatorname{tg^2}x\,$
d)
$\,h(x)\,=\,\operatorname{sen^2}x\,$
e)
$\,h(x)\,=\,\operatorname{sec^2}x\,$
 
 

 



resposta: (C)
×
(MACKENZIE) A função $\,f\,$ definida em $\,\mathbb{R}- \lbrace 2 \rbrace\,$ por $\;f(x)\,= \large{\,\frac{2\,+\,x}{2\,-\,x}\,}\;$ é inversível. O seu contradomínio é $\,\mathbb{R} \,-\,\lbrace a \rbrace\;$. O valor de $\;a\;$ é:
a)
2
b)
-2
c)
1
d)
-1
e)
0

 



resposta: (D)
×
(PUCC - 1982) Sabendo-se que $\,(g \circ f)(x)\,=\, 9x^2\,-\,9x\,+\,2\;$ e que $\;f(x)\,=\,3x\,-\,2\;$, determine $\,g(x)\,$.

 



resposta: $\,g(x)\,=\,x^2\,+\,x\,$
×
(PUCC - 1982) Seja $\;f\,:\, \mathbb{R} \rightarrow \mathbb{R} \;$ uma função definida por $\;f(x)\,=\,4x\,-\,x^2\;$.
Definir a função $\;g(x)\,=\,f(x\,+\,2)\,-\,f(3)\;$.

 



resposta: $\;g(x)\,=\,-x^2\,-\,4x\,+\,5\;$
×
(STA CASA - 1982) Diz-se que uma funçao $\,f\,$ é ímpar se, para todo x de seu domínio, tem-se que $\;f(-x)\,=\,-\,f(x)\;$. Se as funções seguintes são tais que $\;f\,:\,A \subset \mathbb{R} \rightarrow \mathbb{R} \;$, qual delas pode ser ímpar?
a)
$\;f(x)\,=\,x^2\,+\,1\;$
b)
$\;f(x)\,=\,{\large \frac{1}{x}}\;$
c)
$\;f(x)\,=\,\operatorname{log_3}x\phantom{X}$
d)
$\;f(x)\,=\,3x\,-\,1\;$
e)
$\;f(x)\,=\,2^x \,+\,2^{-x}\;$
 
 

 



resposta: (B)
×
(MACKENZIE) Uma funcão $\,f\,$ é definida em $\,A\,$ e tem imagem em $\,B\,$. Sabe-se que o conjunto $\,A\,$ tem 2K - 2 elementos e o conjunto $\,B\,$ tem K + 3 elementos. Se $\,f\,$ é injetora, então:
a)
$\,1 < K \leqslant 5\,$
b)
$\,5 < K \leqslant 7\,$
c)
$\,7 < K < 8\,$
d)
$\,8 < K < 10\,$
e)
$\,K\geqslant 10\,$
 
 

 



resposta: (A)
×
(MACKENZIE - 1982) Seja a função $\;f\,:\, \mathbb{R} \rightarrow \mathbb{R} \;$ definida por $\;f(x)\,=\,3\;$.
Então $\;g\,:\, \mathbb{R} \rightarrow \mathbb{R} \;$ definida por

$\;g(x)\,=\,$$ \; \underbrace{f(x)\centerdot f(x)\centerdot f(x)\centerdot f(x)\, ...\, f(x)}_{\large n \, fatores \, iguais \, a \,f(x)}\;$

será:
a)
ímpar, para todo n
b)
ímpar, só para n ímpar
c)
par, para todo n
d)
par, só para n par
e)
nenhuma das anteriores está correta

 



resposta: (C)
×
Na figura, calcular $\,h\;$ e $\,d\,$.
triângulo retângulo 30 60 graus

 



resposta: Resolução:
$\,\triangle BCD \left\{ \operatorname{tg}60^o \,=\,{\large \frac{h}{d}} \; \Rightarrow \; h\,=\,d\sqrt{3} \right.\,$
$\,\triangle ACD \left\{ \operatorname{tg}30^o \,=\,{\large \frac{h}{d\,+\,40}} \; \Rightarrow \; h\,=\,\frac{\sqrt{3}(d\,+\,40)}{3} \right.\,$
Então $\,d\sqrt{3}\,=\,\frac{\sqrt{3}(d\,+\,40)}{3} \,\Rightarrow\; d\,=\,20\,m$
e portanto $\;h\,=\,20\sqrt{3}\,m\,$

Resposta: $\; \boxed{ d\,=\,20\,m}\;\;\boxed{h\,=\,20\sqrt{3}\,m}$
×
Sabendo-se que $\;\hat{x}\;$ é um ângulo agudo e que $\;\operatorname{tg}\hat{x}\,=\,{\large \frac{5}{12}}\;$, calcule o $\,\operatorname{sen}\hat{x}\,$

 



resposta: Resolução:
$\,\operatorname{sen^2}x \,=\,{\large \frac{\operatorname{tg^2}x}{1\,+\,\operatorname{tg^2}x}}\; \Rightarrow \operatorname{sen^2}x \,=\,{\large \frac{\frac{25}{144}}{1\,+\,\frac{25}{144}}} \,=\,\frac{25}{169}$
Então $\,\boxed{\operatorname{sen}x\,=\,\frac{5}{13}}\;\text{ (para x agudo) }$
×
Calcular $\,y\,=\,{\Large \frac{\operatorname{cos}x\,-\,\operatorname{sec}x}{\operatorname{sen}x\,-\,\operatorname{cossec}x}}\;$, sabendo que $\,\operatorname{tg}x\,=\,3\;$.

 



resposta: Resolução:
$\,y\,=\, {\large \frac{\operatorname{cos}x\,-\,\frac{1}{\operatorname{cos}x}}{\operatorname{sen}x\,-\,\frac{1}{\operatorname{sen}x } }}\,=\, {\Large \frac{ \frac{ \operatorname{cos^2}x\,-\,1}{\operatorname{cos}x}}{\frac{\operatorname{sen^2}\,-\,1}{\operatorname{sen}x}} }\,=\,$ $ {\Large \frac{ - \frac{\operatorname{sen^2}x}{\operatorname{cos}x} } {- \frac{\operatorname{cos^2}x }{\operatorname{sen}x } } } \,=\,$ $ {\Large \frac{\operatorname{sen^3}x }{\operatorname{cos^3}x} \,=\,\operatorname{tg^3}x}$
Então $\,\boxed{y\,=\,3^3\,=\,27}\,$
×
Simplificar a expressão: $\,y\,=\,{\large \frac{\operatorname{cos^3}a \,-\,\operatorname{sen^3}a}{1\,+\,\operatorname{sen}a \;\centerdot\; \operatorname{cos}a } }\;$.

 



resposta: Resolução:
$\,y\,=\,{\large \frac{(\operatorname{cos}a - \operatorname{sen}a)(\operatorname{cos^2}a\,+\,\operatorname{cos}a \;\centerdot\; \operatorname{sen}a\,+\operatorname{sen^2}a)}{(1\,+\,\operatorname{sen}a \;\centerdot\; \operatorname{cos}a)} } \,=\,$
$\,=\,{\large \frac{(\operatorname{cos}a\,-\,\operatorname{sen}a)(1\,+\,\operatorname{sen}a \;\centerdot\; \operatorname{cos}a)}{(1\,+\,\operatorname{sen}a \;\centerdot\; \operatorname{cos}a)}}\,=\,\boxed{\operatorname{cos}a\,-\,\operatorname{sen}a}$

×
(PUC) Qual é o valor de$\phantom{X}{\large x}\phantom{X}$na figura ao lado?
a)
${\large\frac{\sqrt{2}}{3}}$
b)
${\large\frac{5\sqrt{3}}{3}}$
c)
${\large\frac{10\sqrt{3}}{3}}$
d)
${\large\frac{15\sqrt{3}}{4}}$
e)
${\large\frac{20\sqrt{3}}{3}}$
triângulo com 30 e 60 graus

 



resposta: (E)
×
Determine o vértice e o conjunto imagem da função $\;f\;\text{ de }\,\mathbb{R}\,\text{ em } \,\mathbb{R}\;$ definida por $\;f(x)\,=\,2x^2 \,-\,12x\,+\,10\;$.

 



resposta: Vértice: $\,V\,=\,(3;\,-8)\;$
Conjunto Imagem: $\;Im(f)\,=\,[-8;\,+\infty[ \;$ ou $\;Im(f)\,=\,\lbrace \,y\in \mathbb{R} \mid \; y \geqslant -8 \,\rbrace$
×
(MAUÁ) Determinar a equação da parábola que tem seu eixo paralelo ao eixo $\;y\;$, tangencia o eixo $\;x\;$ no ponto $\;V(-1,\,0)\;$ e corta o eixo $\;y\;$ no ponto $\;P(0;\,1)\;$.

 



resposta: $\;f(x)\,=\,x^2\,+\,2x\,+\,1\;$
×
$\;x^2\,-\,mx\,+\,9 \, > \, 0 \;\text{, }\; \vee \negthickspace \negthickspace \negthickspace \negthinspace - x \,\in\, \, \mathbb{R}\,$, se e somente se:
a)
$\;-6 \leqslant m \leqslant 6\;$
b)
$\;-6 < m < 6\;$
c)
$\;m < -6 \;\text{ ou }\; m > 6\;$
d)
$\;m \leqslant -6 \;\text{ ou }\; m \geqslant 2\;$
e)
$\;m < -2 \;\text{ ou }\; m > 6$

 



resposta: (B)
×
Se 30° é raiz da equação $\phantom{X}\operatorname{sen^2}x\,+\,m\centerdot\operatorname{cos^2}x\,=\,\operatorname{tg^2}x\phantom{X}$, então:
a)
m = 1
b)
m = 1/9
c)
m = 0
d)
m = -1
e)
m = 3/2

 



resposta: (B)
×
Assinale (V) ou (F) conforme as sentenças sejam verdadeiras ou falsas:

Se $\;f\;$ é uma função de $\;{\rm I\!N}\;\text{ em }\;{\rm I\!R}\;$ definida por $\;f(x)\,=\,a_x\;$,
com $\;x\in {\rm I\!N}^* \; \text{ e }\; a_x \in {\rm I\!R}\;$, então:

( )
a)
$\;f\;$ é uma sequência de números reais.
( )
b)
$\;D(f)\,=\,{\rm I\!N}^* \; \text{ e }\; CD(f)\,=\,{\rm I\!R}$
( )
c)
pode-se representar $\;f\,=\,(a_n)\,=$ $\;(a_1,\,a_2,\,a_3,\,a_4,...\,,a_n,\,...),$ $\;n\in{\rm I\!N}^*\;$.
( )
d)
$\;(a_n)\;$ é estritamente crescente se, e somente se, $\;a_n < a_{n + 1}\;\text{, }$ $\;\vee \negthickspace \negthickspace \negthickspace \negthinspace - n \,\in\, \, {\rm I\!N}^*\,$.
( )
e)
$\;(a_n)\;$ é estritamente decrescente se, e somente se, $\;a_n > a_{n + 1}\;\text{, }$ $\;\vee \negthickspace \negthickspace \negthickspace \negthinspace - n \,\in\, \, {\rm I\!N}^*\,$.
( )
f)
$\;(a_n)\;$ é constante se, e somente se, $\;a_n\,=\,a_{n+1}\;\text{, }\vee \negthickspace \negthickspace \negthickspace \negthinspace - n \,\in\, \, {\rm I\!N}^*\,$
( )
g)
$\;(a_n)\;$ é crescente se, e somente se, $\;a_n\,\leqslant\,a_{n+1}\;\text{, }\vee \negthickspace \negthickspace \negthickspace \negthinspace - n \,\in\, \, {\rm I\!N}^*\,$
( )
h)
$\;(a_n)\;$ é decrescente se, e somente se, $\;a_n\,\geqslant\,a_{n+1}\;\text{, }\vee \negthickspace \negthickspace \negthickspace \negthinspace - n \,\in\, \, {\rm I\!N}^*\,$
( )
i)
$\;(a_n)\;$ é alternante se, e somente se, $\;a_n\;$ não é monotônica.

 



resposta:
todas corretas

×
Determine a sentença que define a função polinomial do 2º grau cuja representação gráfica é:
gráfico da função f de x do segundo grau

 



resposta: $\;f(x)\,=\,{\large\frac{3x^2}{4}} \,-\,3x\;$

×
Seja $\;f\,:\, {\rm I\!R} \rightarrow {\rm I\!R} \;$ uma função polinomial do 4° grau cujo gráfico é:
gráfico da função polinomial do quarto grau
Determinar o conjunto verdade de:
a)
f(x) = 0
b)
f(x) > 0
c)
f(x) < 0

 



resposta: Resolução:
a) O conjunto verdade para f(x) = 0 é o conjunto de valores para os quais y = 0. Observando o gráfico, y = 0 quando x é igual a -3 , 1 e 4. Portanto se $\,f(x)\,=\,0\,$ o conjunto verdade é
$\,V\,=\,\lbrace -3,\,1,\,4 \rbrace\,$.
b) O conjunto verdade de f(x) > 0 é o conjunto de todos os valores de x que correspondem a um y positivo e diferente de zero, a saber x < -3, x > 1 e x < 4 , e finalmente x >4. Então $\,V\,=\,\lbrace x\,\in\,\mathbb{R} \mid \,x < -3 \; \text{ ou } \; x > 1\;\text{ e }\; x\,\neq\,4\,\rbrace\,$.
c) O conjunto verdade de f(x) < 0 é o conjunto de valores para os quais y < 0, ou seja, verificando no gráfico, x é maior que -3 e menor que 1. $\,V\,=\,\lbrace x\,\in\,\mathbb{R} \mid \,-3 < x < 1\,\rbrace\,$.
×
Resolver em $\,\mathbb{R}\,$ as inequações, aplicando as propriedades da desigualdade.
a)
$\,3x\,-\,6\,<\,0\,$
b)
$\,-3x\,+\,6\,<\,0\,$
c)
$\,6\,-\,2x\,\geqslant\,0\,$
d)
$\,x\,-\,3\,<\,x\,+\,3\,$
e)
$\,-x\,+3\,\leqslant \,x\,+\,3\,$
f)
$\,x\,-\,2\, > \,x\,+\,2\,$

 



resposta: Resolução:
a)
$\,3x\,-\,6\,<\,0\;\Rightarrow $ $ \; 3x\,<\,6\; \Rightarrow $ $ \;\boxed{x<2}\,$
b)
$\,-3x\,+\,6\, < \, 0 \; \Rightarrow $ $ \; -3x\, < \, -6 \;\Rightarrow $ $ \; \boxed{x > 2} \,$
c)
$\,6\,-\,2x\,\geqslant 0 \; \Rightarrow $ $ \; -2x\, \geqslant \,-6 \;\Rightarrow $ $ \boxed{x \leqslant 3}\,$
d)
$\,x\,-\,3\, < \, x\,+\,3 \; \Rightarrow $ $ \; 0x\, < 6 \;$ que ocorre para $\; \boxed{\,\vee \negthickspace \negthickspace \negthickspace \negthinspace - x \,\in\, \, \mathbb{R} \,}\,$
e)
$\,-x\,+\,3\,\leqslant \,x\,+\,3\; \Rightarrow $ $ \,-2x \, \leqslant \, 0 \Rightarrow $ $ \boxed{x \geqslant 0}\,$
f)
$\,x\,-\,2\, > \, x\,+\,2 \; \Rightarrow $ $ \; 0x \, > \, 4 \; \Rightarrow $ $ \; \boxed{x \notin \mathbb{R}}\;$ ou $ \; \mathbb{S} \,=\, \varnothing \,$

×
(PUC) Seja $\;x\;$ elemento de $\;\mathbb{A}\;$. Se $\;x\,\notin \;]{\small -1};\,2]\,\text{, }\; x < 0\;$ ou $\; x \geqslant 3\,$, determine $\;\mathbb{A}\,$.

 



resposta: $\,A\,=\,\lbrace x\,\in\,\mathbb{R} \mid \,x\,\leqslant \, -1 \; \text{ ou } \;x\, \geqslant \, 3 \,\rbrace\,$

×
Na figura, as curvas tracejada e cheia são os gráficos das funções $\,f\,$ e $\,g\,$, respectivamente.
funções efe e gê
São feitas as afirmações a seguir de (I) a (V):
Os únicos valores de $\,x \, \in \; [{\small -3};\,5]\;$ tais que:
I)
$\,f(x)\,=\,0\;$ são $\; x\,=\,-2\;$ ou $\;x\,=\,3\,$
II)
$\,f(x)\, > \, 0 \;$ são $ \; -2 \, < \,x\, < \, 5\,$
III)
$\,f(x)\, \geqslant \, 0 \;$ são $ \; -2 \, \leqslant \,x\, \leqslant \, 5\,$
IV)
$\,f(x)\, < \, 0 \;$ são $\; -3 \, < \,x\,\leqslant \, -2\,$
V)
$\,f(x)\, \leqslant \, 0 \;$ são $ \; -3 \, \leqslant \,x\, \leqslant \, -2\,$
Responda de acordo com o código:
a)
Se todas as afirmações estão corretas
b)
Se apenas (I) e (III) estão corretas
c)
Se apenas (II) e (IV) estão corretas
d)
Se apenas (I) e (V) estão corretas
e)
Se todas estão erradas

 



resposta: (B)
×
Na figura, as curvas tracejada e cheia são os gráficos das funções $\,f\,$ e $\,g\,$, respectivamente.
funções efe e gê
São feitas as afirmações a seguir de (I) a (V):
Os únicos valores de $\,x \, \in \; [{\small -3};\,5]\;$ tais que:
I)
$\,g(x)\,=\,0\;$ são $\; x\,=\,-2\;$ ou $\;x\,=\,1\;$ ou $\;x\,=\,4$
II)
$\,g(x)\, > \, 0 \;$ são $ \; 1 \, < \,x\, < \, 5\,$
III)
$\,g(x)\, \geqslant \, 0 \;$ são $\; 4 \,\leqslant \,x\, \leqslant \, 5\,$
IV)
$\,g(x)\, < \, 0 \;$ são $\; -2 \, \leqslant \,x\,\leqslant \, 1\,$
V)
$\,g(x)\, \leqslant \, 0 \;$ são $ \; -2 \, \leqslant \,x\, \leqslant \, 1\;$ ou $\;x\,=\,4$
Responda de acordo com o código:
a)
Se todas as afirmações estão corretas
b)
Se apenas (I) e (III) estão corretas
c)
Se apenas (II) e (IV) estão corretas
d)
Se apenas (I) e (V) estão corretas
e)
Se todas estão erradas

 



resposta: (D)
×
Veja exercÍcio sobre:
trigonometria
funções trigonométricas
função seno